Q5

 
canylaw
Thanks Received: 0
Vinny Gambini
Vinny Gambini
 
Posts: 21
Joined: July 24th, 2011
 
 
 

Q5

by canylaw Mon Sep 12, 2011 1:33 pm

Why answer A is the correct AC?
This game got me. :x
 
robowarren
Thanks Received: 0
Jackie Chiles
Jackie Chiles
 
Posts: 26
Joined: October 19th, 2011
 
 
 

Re: Q5

by robowarren Sun Feb 05, 2012 1:10 pm

Hi,

I thought the answer here was D - I just looked at the answer key.

Is it not?
 
timmydoeslsat
Thanks Received: 887
Atticus Finch
Atticus Finch
 
Posts: 1136
Joined: June 20th, 2011
 
 
trophy
Most Thanked
trophy
First Responder
 

Re: Q5

by timmydoeslsat Sun Feb 05, 2012 6:46 pm

It is D.

If K were to be the only person other than the driver in a car, we would have this:

1) _ K

2) _ _ _ _

We know that the GL block will go in 2. We know that since J must ride with either F or K as the driver, that J will go with F in car 2, with F being the driver. Since G cannot driver this #2 car, H has to be with F. We now have 5 occupants in car 2 and just one in car 1. This cannot be true because we know each car must have at least 2 occupants.
User avatar
 
tommywallach
Thanks Received: 468
Atticus Finch
Atticus Finch
 
Posts: 1041
Joined: August 11th, 2009
 
 
 

Re: Q5

by tommywallach Mon Aug 06, 2012 2:55 pm

5. (D)
Question Type: Unconditional

We have returned to the world of unconditional questions. With no new constraints, we have only our old diagram to go by. The question is asking us which answer could never be true, which means we’ll have to try and find a situation where each answer choice COULD be true in order to eliminate it.

(A) Well we already know that Gus has to be with Lisa, so this would require Lisa to be driving. Is that possible? Yes! Faith could drive the other car, bringing Hannah, Juan, and Kenneth with her.

(B) We know that either Faith or Gus has to drive Hannah (Constraint 1). If Gus drives her, he also has to bring Lisa (Constraint 3), which is no good. But if Faith drives her, everything is fine. Kenneth could drive the other car (to meet Constraint 2), and bring the other three workers with him.

(C) We know that either Faith or Kenneth has to drive Juan (Constraint 2). If Faith drives Juan, Gus could take all the other workers in his car. If Kenneth drives Juan, Faith could take all the other workers in her car. Either way, we can get Juan alone with his driver.

(D) If Kenneth doesn’t drive, he must be driven by someone, and we already know that neither Hannah nor Juan can drive. If Kenneth is driven by Faith, Juan must come along (by Constraint 2), which is no good. If he’s driven by Gus, Lisa hs to come along (by Constraint 3), which is no good. Finally, if he’s driven by Lisa, Gus has to come along (also by Constraint 3). There’s no way for Kenneth to be alone with his driver.

(E) We already know this is possible from (A).
Tommy Wallach
Manhattan LSAT Instructor
twallach@manhattanprep.com
Image
 
JenniferK632
Thanks Received: 0
Jackie Chiles
Jackie Chiles
 
Posts: 43
Joined: January 18th, 2020
 
 
 

Re: Q5

by JenniferK632 Sat Jul 18, 2020 9:03 am

I ended up getting this question correct, but in timed circumstances I would have compromised solving other questions. Any advice on could be true questions? Gosh, this was very time consuming for me! Thanks.
User avatar
 
smiller
Thanks Received: 73
Atticus Finch
Atticus Finch
 
Posts: 205
Joined: February 01st, 2013
 
 
 

Re: Q5

by smiller Mon Jul 20, 2020 2:55 am

Have you tried using work from other questions? Since this is asking for an answer that CANNOT be true, we can eliminate answers that show up in any valid hypothetical. For example, the diagram that we create to answer question #3 in this game shows that Gus is able to be alone with Lisa in a car, with Lisa driving. This allows us to quickly eliminate answer choice (A) in question #5. You might not be able to get all the way to the right answer using this technique. You might still need to test a couple of answers. But using other work to eliminate even one or two answers quickly can save quite a bit of time.